How can I solve for W in this tricky statics problem without exceeding 1050N?

In summary, you have trouble with a HW question that has been driving you crazy for a few days now. You have a free body diagram and found the slope to be 8/5, which causes T3 and T2 to be directly in line with each other. This fact threw you off, so you tried several different approaches and each led you to an incorrect answer. The book gives a range for W of 0 < W < 609N, and you have found that T2 will be 1050N when the W is 609N.
  • #1
paul11273
156
0
I am having trouble with a HW question that has been driving me crazy for a few days now. I have to solve for W ( a hanging weight) that will not cause either T1 or T2 to exceed 1050N. So far I have drawn the free body diagram that is attached as a word doc, and the angles I found based on the dimensions given. The problem gives T3 as 680N and shows the slope to be 8/5. This causes (I think) T3 and T2 to be directly in line with each other. I feel like I am letting this fact throw me off. Also, when I start trying to break up each one into component vectors, it seems that I have too many unknowns. I have tried several different approaches but each leads me to an incorrect answer. The book gives a range for W of 0 < W < 609N.
Can you help me figure this one out? Atleast lead me in the right direction? Thanks.
 

Attachments

  • hw 2.57.doc
    24.5 KB · Views: 288
Physics news on Phys.org
  • #2
These are the two equations:

1. cos(28)T2 - cos(53)T1 = cos(28)680
2. sin(53)T1 + sin(28)T2 - sin(28)680 = W

You are trying to find W where T1 or T2 is equal to 1050. So substitute 1050 in for T1 or T2, and solve for the other T. Then solve for the weight.
 
  • #3
Thanks!
I can't believe that I killed myself over this question. I had those two equations, but failed to see that I need to sub in 1050 for either T1 or T2, then solve for the other.
I have completed it, and found that T2 will be 1050N when the W is 609N, and T1 will 544N.
I let T3 confuse me, and it had me thinking the problem would be more complicated.
Thanks again. You have sold me on this forum. I will check here regularly, and hopefully I can contribute as well.
 
  • #4
You have 3 unknowns: T1, T2, and W
You have 2 equations, as Moose pointed out.
You also have 2 inequalities, one of which you must "max out", and the other of which you must ensure holds. Pick one of the inequalities, arbitrarily, and max it out (turn it into an equality). This fixes one of your unknowns (T1 or T2, whichever you chose). Then, solve the equations, and afterwards make sure the other inequality holds. If it does not, then just go back and max out the other inequality and solve.
 

Related to How can I solve for W in this tricky statics problem without exceeding 1050N?

1. What is statics?

Statics is the branch of mechanics that deals with the study of objects at rest or in equilibrium. It involves analyzing the forces acting on an object and determining the resulting motion or lack thereof.

2. Why do students often struggle with statics problems?

Students may struggle with statics problems because they require a strong understanding of mathematical concepts such as vectors and forces. Additionally, statics problems can be complex and involve multiple steps to reach a solution.

3. How can I improve my skills in solving statics problems?

Practice is key when it comes to solving statics problems. It is important to review and understand the fundamental principles and equations involved, and then work through various practice problems to reinforce your understanding.

4. What are some common mistakes to avoid when solving statics problems?

Some common mistakes to avoid when solving statics problems include forgetting to consider all the forces acting on an object, using incorrect units, and not drawing accurate free-body diagrams.

5. How can I approach a statics problem if I am stuck?

If you are stuck on a statics problem, try breaking it down into smaller, more manageable parts. Make sure you have a clear understanding of the given information and what is being asked. You can also try working backwards from the desired solution to identify any mistakes in your approach.

Similar threads

  • Introductory Physics Homework Help
Replies
11
Views
1K
  • Engineering and Comp Sci Homework Help
Replies
16
Views
3K
  • Engineering and Comp Sci Homework Help
Replies
4
Views
1K
  • Introductory Physics Homework Help
Replies
2
Views
870
Replies
17
Views
2K
  • Introductory Physics Homework Help
Replies
7
Views
3K
  • Advanced Physics Homework Help
Replies
1
Views
775
  • Introductory Physics Homework Help
Replies
2
Views
1K
  • Engineering and Comp Sci Homework Help
Replies
1
Views
2K
  • Introductory Physics Homework Help
Replies
4
Views
3K
Back
Top